Double integral over triangular region

Click For Summary
The discussion focuses on evaluating the double integral of the function f(u,v) = v - sqrt(u) over a triangular region defined by u + v = 64 in the first quadrant. The limits of integration are established as 0 ≤ u ≤ 64 and 0 ≤ v ≤ 64 - u. Participants express frustration over the complexity of the calculations, with one user arriving at a fraction of 4638576/30, while another suggests the correct answer is approximately 35,000. The exact value is confirmed as 524288/15, highlighting the challenge posed by the "ugly" numbers involved in the problem. The discussion emphasizes the importance of careful evaluation and simplification in solving such integrals.
anniecvc
Messages
28
Reaction score
0

Homework Statement


Integrate f(u,v)= v - sqrt(u) over the triangular region cut from the first quadrant by the line u+v=64 in the uv plane.

Homework Equations


I am assuming u is the equivalent of the x-axis in the xy plane and v the equivalent of y in the xy plane.
I am taking the triangle as a Type I region.

The Attempt at a Solution


limits of integration:

0≤u≤65, 0≤v≤64-u

∫ ∫ v-sqrt(u) dvdu

∫ (1/2)v2 - sqrt(u)v evaluated from v=0 to v=64-u

∫ 2048 - 64u + (1/2)u2 - 64*u1/2 + u3/2

2048u - 32u2 + (1/6)u3 - (2/3)*64*u3/2+(2/5)*u5/2 evaluated from u=0 to u=64

I get a really narley fraction, namely, 4638576/30, which is of course wrong.
 
Physics news on Phys.org
Everything looks good, I double checked your calculations and came to a different result. Double check the final evaluation. I came up with ~ 8,000 (the exact value is for you to figure out).
 
  • Like
Likes 1 person
Jufro said:
Everything looks good, I double checked your calculations and came to a different result. Double check the final evaluation. I came up with ~ 8,000 (the exact value is for you to figure out).
I get more like 32000.
 
Yes, the exact answer is 524288/15 ~ 35000.

This was a terrible problem not because it was difficult but because the numbers were so ugly.
 
anniecvc said:
Yes, the exact answer is 524288/15 ~ 35000.

This was a terrible problem not because it was difficult but because the numbers were so ugly.

It's not so bad if you leave as much as possible in powers of 2.
211u - 25u2 + (1/6)u3 - (2/3)*26*u3/2+(2/5)*u5/2 where u = 26:
217 - 217 + (1/6)218 - (2/3)*26*29+(2/5)*215 = 216{(2/3) - (1/3)+(1/5)} = 219/15
 
  • Like
Likes 1 person
Question: A clock's minute hand has length 4 and its hour hand has length 3. What is the distance between the tips at the moment when it is increasing most rapidly?(Putnam Exam Question) Answer: Making assumption that both the hands moves at constant angular velocities, the answer is ## \sqrt{7} .## But don't you think this assumption is somewhat doubtful and wrong?

Similar threads

  • · Replies 3 ·
Replies
3
Views
2K
Replies
10
Views
2K
  • · Replies 16 ·
Replies
16
Views
2K
Replies
5
Views
2K
  • · Replies 1 ·
Replies
1
Views
2K
  • · Replies 19 ·
Replies
19
Views
2K
  • · Replies 27 ·
Replies
27
Views
3K
  • · Replies 21 ·
Replies
21
Views
2K
Replies
12
Views
2K
  • · Replies 1 ·
Replies
1
Views
1K